độ cứng xấp xỉ số màu trong đồ thị với mức độ giới hạn


12

Tôi đang tìm kiếm kết quả độ cứng trên màu đỉnh của đồ thị với mức độ giới hạn.

Cho một đồ thị , chúng ta biết rằng đối với bất kỳ ε > 0 , thật khó để ước tính χ ( G ) trong vòng hệ số | V | 1 - ϵ trừ khi NP = ZPP [ 1 ]. Nhưng nếu mức G tối đa bị giới hạn bởi d thì sao? Có bất kỳ tỷ lệ độ cứng của mẫu d 1 - ε (đối với một số εG(V,E)ϵ>0χ(G)|V|1ϵNP=ZPPGdd1ϵϵ ) trong trường hợp này?

Một câu hỏi dễ hơn là: Độ cứng của xấp xỉ số sắc độ cạnh của siêu đồ thị khi kích thước cạnh của chúng được giới hạn bởi . Chúng ta có thể hy vọng cho một d 1 - ε tỷ lệ độ cứng trong trường hợp này? (nói, đối với bất kỳ ε > 0 )dd1ϵϵ>0

Cảm ơn đã quan tâm!


3
bạn có thể đệm một ví dụ cứng với các đỉnh bị cô lập
Sasho Nikolov

2
Có, nhưng nếu bạn đặt một giới hạn hữu hạn vào kích thước của thể hiện cứng mà bạn bắt đầu, nó sẽ không còn khó nữa.
David Eppstein

1
@Sasho Làm thế nào các đỉnh bị cô lập có thể giúp đỡ khi chúng không tăng số lượng màu sắc cũng như mức độ tối đa?
afshi7n

2
@DavidEppstein chắc chắn, phần đệm này chỉ chứng minh điều gì đó nếu d vẫn liên quan đến đa thức. OP, đó thực sự là chính xác điểm. bạn bắt đầu với một ví dụ với d đỉnh (độ để tối đa tối đa là d ) mà rất khó để ước tính χ để trong vòng d 1 - ε . thêm n - d đỉnh bị cô lập. χ ở lại với việc nghỉ cùng và max độ d . đây là polytime nếu N = d O ( 1 ) . vì vậy với bất kỳ số nguyên kndddχd1ϵndχdN=dO(1)k, Có tồn tại trường hợp với tối đa độ mà nó khó có thể xấp xỉ χ để trong vòng d 1 - εd=n1/kχd1ϵ
Sasho Nikolov

Câu trả lời:


9

Như David đã chỉ ra, bài báo của Khot, "Cải thiện kết quả không thể đạt được cho MaxClique, Số màu và Màu đồ thị gần đúng", Định lý 1.6, cho biết đó là đồ thị colorable K- colorable với 2 màu Ω ( ( log K ) 2 ) cho đồ thị có độ lớn nhất là 2 2 ( log K ) 2 , cho hằng số K đủ lớn . Nói cách khác, đối với đồ thị của độ d , rất khó để màu 2 K2Ω((logK)2)22(logK)2Kd graph -colorable vớilogdmàu sắc.2loglogdlogd

Để có được mức độ ràng buộc tốt hơn, có lẽ bạn có thể sử dụng các ý tưởng từ bài báo của Trevisan "Kết quả không gần đúng cho các vấn đề tối ưu hóa trong các trường hợp mức độ giới hạn". Quan sát chính là đồ thị được tạo ra bởi quá trình khử FGLSS là một tập hợp các sơ đồ con lưỡng cực hoàn chỉnh, và người ta có thể thay thế từng đồ thị bằng một bộ phân tán lưỡng cực, là phần thưa hơn. Ý tưởng tương tự được sử dụng trong nhiều kết quả như Chan http://eccc.hpi-web.de/report/2012/110/ , Định lý 1.4 / Phụ lục D.

Tôi nghĩ rằng điều này sẽ cung cấp cho bạn một cái gì đó như thế nào cho đồ thị mức độ màu được giới hạn bởid, rất khó để tô màu nó với màudccho một số không đổi0<c<12clogdddc0<c<1 .

Mức độ ràng buộc trong bài báo mà Michael đề cập tương tự như của Khot, cụ thể là theo cấp số nhân của trường hợp âm thanh. Tất nhiên, cách tiếp cận thưa thớt ở trên cũng cải thiện điều này, nhưng có lẽ sẽ không cung cấp hằng số tốt hơn cho mục đích của bạn.


Cảm ơn bạn đã trả lời hữu ích, Sangxia. Vì vậy, từ giấy của Khot, chúng ta có thể ngụ ý tỷ lệ độ cứng . Tôi nghĩ rằng sử dụng những cải tiến trong giấy của bạn, chúng tôi có thể cải thiện rằng tỷ lệ độ cứng để 2 2 Ω ( 2Ω(loglogd). Đúng không? 22Ω(loglogd)
afshi7n

@ afshi7n Các thông số có một chút khó khăn ở đây. Nói về mức độ, giấy khọt của cho . Bài viết của tôi đưa ra khoảnglogd/(loglogd)3. Chúng tôi có thể cải thiện mức độ của biểu đồ trong việc giảm theo phương pháp của Trevisan. Tôi tin rằng mang lại cho bạndc. Tất cả những điều này đòi hỏi một hằng số đủ lớnd. logd/2loglogdlogd/(loglogd)3dcd
sangxia

1
Tôi hiểu rồi, cảm ơn! Tôi cũng hỏi khọt qua email, ông gọi tôi đến bài viết này siam.org/proceedings/soda/2011/SODA11_124_guruswamiv.pdf mà tôi tin cung cấp cho các giả 2-1 phỏng đoán khọt của. dc
afshi7n

8

Độ cứng được biết đến nhiều nhất của việc xấp xỉ số lượng sắc độ của đồ thị màu với mức độ tối đa giới hạn là do Venkatesan Guruswami và Sanjeev Khanna, Về độ cứng của 4 màu Vẽ đồ thị 3 màu :3

Có là một hằng số như vậy mà đưa ra một 3 graph -colorable với mức độ tối đa nhất Δ , nó là NP-khó để tô màu nó chỉ sử dụng 4 màu sắc.Δ3Δ4


8

Có một kết quả không thể gần đúng để tô màu các đồ thị mức độ giới hạn trong bài báo FOCS'01 của Khot, "Cải thiện kết quả không thể đạt được cho MaxClique, Số màu và Màu đồ thị gần đúng" - có lẽ nó yếu hơn bạn muốn, nhưng ít nhất nó có thể đi đúng hướng.

Ông chứng minh rằng, đối với một tham số (giả định là không đổi), và cho k đồ thị -chromatic với mức độ 2 k O ( log k ) , nó là NP-khó tìm Chất tạo màu rằng việc sử dụng exp ( ( log k ) 2 / 25 ) màu sắc. Vì vậy, về mức độ d , khó có thể tô màu trong một yếu tố O ( log d ) , nhưng tỷ lệ không tương đương cũng là một hàm siêu đa thức của số màu.kk2kO(logk)exp((logk)2/25)dO(logd)


David, cảm ơn bạn đã trả lời. Có, tôi đã thấy kết quả của họ, nhưng tôi hy vọng sẽ có được tỷ lệ độ cứng tốt hơn . Tôi nghĩ rằng điều này có thể dễ dàng đạt được hơn trong vấn đề thứ hai, tức là xấp xỉ số lượng sắc độ cạnh của siêu đồ thị ..logd
afshi7n

Tại sao không hỏi Khốt?
Chandra Chekuri

1
@ Vendra Chỉ cần gửi email và hỏi anh ấy, cảm ơn vì lời đề nghị! Tôi sẽ cập nhật ở đây nếu tôi nghe lại.
afshi7n

klogk/25exp((klogk)/25)2k1/3

k(logk)/25exp((klogk)/25)

4

Kết quả này có thể hữu ích:

Emden-Weinert, Hougardy và Kreuter đã chứng minh rằng việc xác định xem một đồ thị có mức độ tối đa Δ có một màu sử dụng k= =Δ-Δ+1 màu sắc là NP-hoàn chỉnh (k3)

T. Emden-Weinert, S. Hougardy, B. Kreuter, đồ thị có thể tạo được duy nhất và độ cứng của đồ thị màu của chu vi lớn, Combin. Con mồi Tính toán. 7 (4) (1998) 375 Từ386

Khi sử dụng trang web của chúng tôi, bạn xác nhận rằng bạn đã đọc và hiểu Chính sách cookieChính sách bảo mật của chúng tôi.
Licensed under cc by-sa 3.0 with attribution required.